Difference between revisions of "1950 AHSME Problems/Problem 40"

m (Problem)
(Solution)
Line 10: Line 10:
  
 
==Solution==
 
==Solution==
Limits do not take the value of the limiting function at the specified value into account, so we are essentially being asked to find the limit of <math>x+1</math> as <math>x</math> approaches <math>1</math>. This is simply <math>\boxed{\textbf{(D)}\ 2}</math>.
+
Both <math>x^2-1</math> and <math>x-1</math> approach 0 as <math>x</math> approaches <math>1</math>, using the L'Hôpital's rule, we have <math>\lim \limits_{x\to 1}\frac{x^2-1}{x-1} = \lim \limits_{x\to 1}\frac{2x}{1} = 2</math>.
 +
Thus, the answer is <math>\boxed{\textbf{(D)}\ 2}</math>.
 +
 
 +
~ MATH__is__FUN
  
 
==See Also==
 
==See Also==

Revision as of 07:53, 3 December 2023

Problem

The limit of $\frac {x^2-1}{x-1}$ as $x$ approaches $1$ as a limit is:

$\textbf{(A)}\ 0 \qquad \textbf{(B)}\ \text{Indeterminate} \qquad \textbf{(C)}\ x-1 \qquad \textbf{(D)}\ 2 \qquad \textbf{(E)}\ 1$

Solution

Both $x^2-1$ and $x-1$ approach 0 as $x$ approaches $1$, using the L'Hôpital's rule, we have $\lim \limits_{x\to 1}\frac{x^2-1}{x-1} = \lim \limits_{x\to 1}\frac{2x}{1} = 2$. Thus, the answer is $\boxed{\textbf{(D)}\ 2}$.

~ MATH__is__FUN

See Also

1950 AHSC (ProblemsAnswer KeyResources)
Preceded by
Problem 39
Followed by
Problem 41
1 2 3 4 5 6 7 8 9 10 11 12 13 14 15 16 17 18 19 20 21 22 23 24 25 26 27 28 29 30 31 32 33 34 35 36 37 38 39 40 41 42 43 44 45 46 47 48 49 50
All AHSME Problems and Solutions

The problems on this page are copyrighted by the Mathematical Association of America's American Mathematics Competitions. AMC logo.png